8
$\begingroup$

This looked like an easy exercise, when a friend of mine asked me if I know a way to prove that the decimal representation of $3^k$ always contains a zero for $k\ge k_0$, but the more I think about this question the more I find it difficult.

After some experimentation (using Mathematica), I discovered that all the numbers $3^k$ for $69\le k\le 20000$ do have a zero in there decimal representation, and that this zero does appear among the first 100 digits. This suggests that maybe $k_0=69$.

I hope that some one can help in giving some insight on this question.

$\endgroup$
8
  • 2
    $\begingroup$ My advice is to try the last n decimal digits instead. The function is periodic modulo 10^n for any n. That means if you can find an n such that a long enough string have a zero among the last n digits, then you'll be done. $\endgroup$ Oct 10, 2014 at 12:15
  • 3
    $\begingroup$ @James Cranch: The $5\times 10^{n-2}$ values hit mod $10^n$ are arbitrary initial sequences with one of the $500$ allowed final $4$ digits. Since $3^8 = 6561$, we can find a power of $3$ that ends in $1111...1116561$. For example, $3^{195508}$ ends in $...1116561.$ $\endgroup$ Oct 10, 2014 at 12:38
  • 3
    $\begingroup$ Also, the fact that $\alpha = \log(3)/\log(10)$ is irrational, together with the fact that $\{ n \alpha \}$ is uniformly distributed mod $1$ means that for all positive integers $m$, there is an integer $n(m)$ so that the first $m$ digits of $3^{n(m)}$ are all nonzero. $\endgroup$ Oct 10, 2014 at 12:39
  • 4
    $\begingroup$ So this is a version of a known open problem. The version I heard is about 7's in base 10 expansions of $2^n$. You can with a bit of ingenuity prove that this holds for almost all $n$, but all $n$ is probably really hard. If you believe the digits are random (as they "should" be), then you expect the conjecture to be true by Borel-Cantelli type arguments. $\endgroup$ Oct 10, 2014 at 17:40
  • 3
    $\begingroup$ See also mathoverflow.net/questions/30357/… for a closely related question. $\endgroup$ Oct 10, 2014 at 17:42

0

Your Answer

By clicking “Post Your Answer”, you agree to our terms of service and acknowledge you have read our privacy policy.

Browse other questions tagged or ask your own question.